Proving the Recursive Formula for an Using Induction

  • Thread starter Thread starter IntroAnalysis
  • Start date Start date
  • Tags Tags
    Induction
Click For Summary
The discussion focuses on proving the recursive formula an+1 - an = (-1/2)n(a1 - a0) using mathematical induction. The base case for n=1 is verified, showing that a2 - a1 equals the expected formula. The next step involves assuming the formula holds for k and demonstrating it for k+1, confirming the relationship through algebraic manipulation. The proof also references the Triangle Inequality Theorem to support the argument. Overall, the induction process is successfully outlined, leading to the desired conclusion.
IntroAnalysis
Messages
58
Reaction score
0

Homework Statement


Define an = (an-1 + an-2)/2 for each positive integer ≥ 2. Use induction to show that: an+1 - an = (-1/2)n(a1 -a0)


Homework Equations


First show it is true for base case. Assume if it is true for (k), then show it is true for (k + 1).


The Attempt at a Solution


Base case n=1. Then a2 - a1 = (-1/2)1(a1 - a0) = (a0 - a1)/2
Check: a2 - a1 = (a1 + a0)/2 - a1 = (a1 + a0 - 2a1)/2 = (a0 - a1)/2.

So then do I assume that ak+1 - ak = (-1/2)k(a1 - a0)? Then what?
 
Physics news on Phys.org
so assume it is true for k-1,k as below (note you had the power wrong above)
a_k - a_{k-1} = (-1/2)^k(a_1 - a_0)

Now using that can you show
a_{k+1} - a_{k} = (-1/2)^{k+1}(a_1 - a_0)
 
No, I did not have the power wrong. This is from Introduction to Analysis, Gaughan, Prob. 23.

It says you may want to use induction to show that:
an+1 - an = (-1/2)^n(a1 - a0).
 
Got the induction part:

Assume ak+1 - ak = (-1/2)k(a1-a0), then

ak+2 - ak+1 = (ak+1 + ak)/2 -[(-1/2)k(a1 - a0) + ak]

= (ak+1 + ak - 2ak)/2 - [(-1/2)k(a1 - a0)]
= (ak+1 - ak)/2 - [(-1/2)k(a1 - a0)]
= (1/2)(-1/2)k(a1 - a0) - [(-1/2)k(a1 - a0)]
= -(-1/2)k+1(a1 - a0) - [(-1/2)k(a1 - a0)]
= (-1/2)k(a1 - a0)[1/2 - 1] = (-1/2)k+1(a1 - a0) as desired.

The rest of the proof uses Triangle Inequality Theorem.
 
Last edited:
Question: A clock's minute hand has length 4 and its hour hand has length 3. What is the distance between the tips at the moment when it is increasing most rapidly?(Putnam Exam Question) Answer: Making assumption that both the hands moves at constant angular velocities, the answer is ## \sqrt{7} .## But don't you think this assumption is somewhat doubtful and wrong?

Similar threads

  • · Replies 12 ·
Replies
12
Views
2K
  • · Replies 1 ·
Replies
1
Views
1K
  • · Replies 1 ·
Replies
1
Views
2K
  • · Replies 1 ·
Replies
1
Views
2K
  • · Replies 2 ·
Replies
2
Views
2K
  • · Replies 3 ·
Replies
3
Views
3K
  • · Replies 10 ·
Replies
10
Views
9K
  • · Replies 17 ·
Replies
17
Views
3K
  • · Replies 1 ·
Replies
1
Views
1K
  • · Replies 1 ·
Replies
1
Views
2K